You are on page 1of 67

91).

Since 1981, when the farm depression began, the number of


acres overseen by professional farm-management companies have
grown from 48 million to nearly 59 million, an area that is about
Colorado's size.

(A) have grown from 48 million to nearly 59 million, an area that is


about Colorado's size

(B) have grown from 48 million to nearly 59 million, about the size
of Colorado

(C) has grown from 48 million to nearly 59 million, an area about


the size of Colorado

(D) has grown from 48 million up to nearly 59 million, an area


about the size of Colorado's

(E) has grown from 48 million up to nearly 59 million, about


Colorado's size

92). Some bat caves, like honeybee hives, have residents that take on
different duties such as defending the entrance, acting as sentinels
and to sound a warning at the approach of danger, and scouting
outside the cave for new food and roosting sites.

(A) acting as sentinels and to sound

(B) acting as sentinels and sounding

(C) to act as sentinels and sound

(D) to act as sentinels and to sound

(E) to act as a sentinel sounding

Answers --

91). C is the best choice.


A, B -- incorrect - subject verb agreement - plural verb "have" does
not agree with singular subject "number".
D, E -- incorrect - unidiomatic - "up to" is wrong, correct idiom is
"from x to y"

92). B is the best choice -- maintaining parallelism required among


and within the three main verb phrases - defending, acting...
sounding, and scouting
A, C, D, E -- incorrect - violating parallelism.
POSTED BY PRACHI PAREEKH AT 9:43 AM 0 COMMENTS LINKS TO THIS POST

LABELS: IDIOM, PARALLELISM, SUBJECT VERB AGREEMENT

WEDNESDAY, NOVEMBER 08, 2006

Gmat Sentence Correction 89, 90

89). John Smith provides information on the conditions that lead


women to a gynecologist, and he notes that these conditions can,
and sometimes are, used in the promotion of surgery which is not
needed.

(A) and he notes that these conditions can, and sometimes are, used
in the promotion of surgery which is not needed

(B) noting that these conditions can be, and sometimes are, used to
promote needless surgery

(C) and notes that they could and sometimes are used unnecessarily
in promoting surgery

(D) sometimes promoting needless surgery

(E) which they use sometimes to promote unnecessary surgery

90). Machines replacing human labor, there was wide anticipation


that the workweek would continue to become shorter.

(A) Machines replacing human labor, there was wide anticipation


that

(B) When machines replaced human labor, there was wide


anticipation

(C) As machines replaced human labor, it was widely anticipated


that

(D) Insofar as machines replaced human labor, it was widely


anticipated

(E) Human labor being replaced by machines, there was wide


anticipation that
Answers --

89). B is the best choice.


A - incorrect - wordy...Further "can be (used) and sometimes are
used"..omits "be"
C, E - incorrect - ambiguous "they"
D - incorrect - who promotes needless surgery ?...not
clear....Conditions or John Smith?

90). Best choice C - only choice to use past tense correctly


throughout.
Why It is right in this choice, the link below explains ...

http://gmat-grammar.blogspot.com/2006/07/it-as-subject-of-
sentence.html

POSTED BY PRACHI PAREEKH AT 12:21 AM 0 COMMENTS LINKS TO THIS


POST
LABELS: PRONOUN ERROR, TENSE, WORDY

TUESDAY, NOVEMBER 07, 2006

Gmat Sentence Correction 87, 88

87). It takes a deft balance between savings discipline, investment


knowledge, risk taking, luck, and time to raise a million dollars
through investments.

(A) It takes a deft balance between

(B) Deft balancing is needed between

(C) Deftly balanced, it takes

(D) It takes a deft balance of

(E) A deft balance is what one needs among

88). Last year, land values in most parts of the pinelands rose
almost so fast, and in some parts even faster than what they did
outside the pinelands.

(A) so fast, and in some parts even faster than what they did

(B) so fast, and in some parts even faster than, those


(C) as fast, and in some parts even faster than, those

(D) as fast as, and in some parts even faster than, those

(E) as fast as, and in some parts even faster than what they did

Answers --

87). D is the best choice.


A, B - incorrect - usage of between is wrong here...."between" is used
when we consider only two objects. Further correct idiom is
"balance of"
C - incorrect - Awkward.....Deftly balanced seems to modify the full
sentence
E - incorrect - Changes the meaning (deft balance among a, b and c )
---"among" incorrect here, we need "of" here.
"Deft balance of" is not same as "Deft balance among"
E suggests that "Deft balance" is the only required object among all
the other objects "savings discipline, investment knowledge, risk
taking, luck and time" ...which is incorrect.

88). D - correct - idiomatic


Comparison in this sentence is between the most parts of pineland
and parts outside the pineland.
A, B, C - incorrect - correct idioms are "as...as" and "so..that"
E - incorrect - "they" referring to what ?
POSTED BY PRACHI PAREEKH AT 11:46 PM 0 COMMENTS LINKS TO THIS
POST
LABELS: COMPARISON, IDIOM, PRONOUN ERROR

MONDAY, NOVEMBER 06, 2006

Gmat Sentence Correction 85, 86

85). Defense attorneys have occasionally argued that their clients'


misconduct stemmed from a reaction to something ingested, but in
attributing criminal or delinquent behavior to some food allergy,
the perpetrators are in effect told that they are not responsible for
their actions.

(A) in attributing criminal or delinquent behavior to some food


allergy
(B) if criminal or delinquent behavior is attributed to an allergy to
some food

(C) in attributing behavior that is criminal or delinquent to an


allergy to some food

(D) if some food allergy is attributed as the cause of criminal or


delinquent behavior

(E) in attributing a food allergy as the cause of criminal or


delinquent behavior

86). The voluminous personal papers of Thomas Alva Edison reveal


that his inventions typically sprang to life not in a flash of
inspiration but evolved slowly from previous works.

(A) sprang to life not in a flash of inspiration but evolved slowly

(B) sprang to life not in a flash of inspiration but were slowly


evolved

(C) did not spring to life in a flash of inspiration but evolved slowly

(D) did not spring to life in a flash of inspiration but had slowly
evolved

(E) did not spring to life in a flash of inspiration but they were
slowly evolved

Answers --

85). B is the correct answer.


A, C, E - incorrect - "in attributing criminal or delinquent behavior"
is modifying perpetrators which is not correct because the defense
attorneys are attributing behavior to food allergies.
D - incorrect - doesn't identifies the attributors at all.

86). C is the correct answer - Parallelism - ...not...but..


A, B - incorrect - violating parallelism - right form...not in....but in...
D - incorrect - wrong tense
E - incorrect - passive
POSTED BY PRACHI PAREEKH AT 12:09 AM 1 COMMENTS LINKS TO THIS
POST
LABELS: MODIFIER ERROR, TENSE

SUNDAY, NOVEMBER 05, 2006


Gmat Sentence Correction 83, 84

83). Aho, a Kiowa matriarch, held festivals in her home, they


featured the preparation of great quantities of ceremonial food, the
wearing of many layers of colorful clothing adorned with silver, and
the recounting of traditional tribal jokes and stories.

(A) Aho, a Kiowa matriarch, held festivals in her home, they


featured

(B) Festivals were held in Aho, a Kiowa matriarchs home, which


featured

(C) Aho, who was a Kiowa matriarch in her home, held festivals
featuring

(D) In her home, Aho, a Kiowa matriarch, held festivals that


featured

(E) Aho, a Kiowa matriarch, held festivals in her home that featured

84). An array of tax incentives has led to a boom in the construction


of new office buildings; so abundant has capital been for commercial
real estate that investors regularly scour the country for areas in
which to build.

(A) so abundant has capital been for commercial real estate that

(B) capital has been so abundant for commercial real estate, so that

(C) the abundance of capital for commercial real estate has been
such,

(D) such has the abundance of capital been for commercial real
estate that

(E) such has been an abundance of capital for commercial real


estate,

Answers --

83). D is the best answer.


A - incorrect - referent of "they" is not present or in other words is
unclear
B - incorrect - Aho, a Kiowa matriarchs home, makes it wrong as
Aho is Kiowa matriarch and not the home
C - incorrect - is unnecessarily wordy -- "who was" part
E - incorrect - ambiguous - not clear whether the festival or the home
featured the preparation.

84). Choice A is the correct answer.


B - incorrect - there is repetition of "so" as such it is illogical,
C, E - incorrect - "that" has been omitted which makes both the
choices grammatically incorrect.
D - incorrect - seperation of "has" and "been" is vague and incorrect.
C, D ,E - incorrect - "such" seems to convey the meaning "of a kind"
and it is not stressing "abundant" as it should do otherwise.
POSTED BY PRACHI PAREEKH AT 11:54 PM 0 COMMENTS LINKS TO THIS
POST
LABELS: PRONOUN ERROR, WORDY

Gmat Sentence Correction 81, 82

81). Despite the recent election of a woman to the office of prime


minister, the status of women in Pakistan is little changed from how
it was in the last century.

(A) is little changed from how it was

(B) is a little change from how it was

(C) has changed little

(D) has changed little from how it has been

(E) is little changed from the way it was

82). Although the lesser cornstalk borer is widely distributed,


control of them is necessary only in the South.

(A) the lesser cornstalk borer is widely distributed, control of them


is

(B) widely distributed, measures to control the lesser cornstalk


borer are

(C) widely distributed, lesser cornstalk borer control is

(D) the lesser cornstalk borer is widely distributed, measures to


control it are

(E) it is widely distributed, control of the lesser cornstalk borer is


Answers --

81). C is the best choice.


A, B, D, E - incorrect - "it" referring to what? is it the status of
women in the last century or something else in the last century that
we compare the current women status with.
A, B, E - incorrect - "little changed" is incorrect..(little changed
means changed to some extent but not
much....whereas...changed little implies did not changed at all)
B - incorrect - use of "is" (present tense) is incorrect
D - incorrect - sentence says "last century" so "how it has been" is
unnecessary to state.

82). D is the best choice.


A - incorrect - "them" is refering to "lesser cornstalk borer" so must
be singular.
B - incorrect - "widely distrbuted" is modifying "measures" , should
modify "the lesser cornstalk borer".
C - incorrect - changing the meaning - suggests that lesser cornstalk
borer control is widely distributed instead of lesser cornstalk borer.
E - incorrect - "it" referring to what ? "lesser cornstalk borer"or
"control"
POSTED BY PRACHI PAREEKH AT 11:23 PM 0 COMMENTS LINKS TO THIS
POST
LABELS: MODIFIER ERROR, PRONOUN ERROR, TENSE

TUESDAY, OCTOBER 10, 2006

Gmat Sentence Correction 79, 80

79). Federal incentives now encourage investing capital in


commercial office buildings despite vacancy rates in existing
structures that are exceptionally high and no demand for new
construction.

(A) investing capital in commercial office buildings despite vacancy


rates in existing structures that are exceptionally high and

(B) capital investment in commercial office buildings, even though


vacancy rates in existing structures are exceptionally high and there
is

(C) capital to be invested in commercial office buildings even though


there are exceptionally high vacancy rates in existing structures
with

(D) investing capital in commercial office buildings even though the


vacancy rates are exceptionally high in existing structures with

(E) capital investment in commercial office buildings despite


vacancy rates in existing structures that are exceptionally high, and
although there is

80). A large and increasingly influential sector of publishing, 20


percent of all the National and American Book awards since 1950
have gone to university-press books.

(A) A large and increasingly influential sector of publishing, 20


percent of all the National and American Book awards since 1950
have gone to university-press books.

(B) A large and increasingly influential sector of publishing,


university-press books have won 20 percent of all the National and
American Book awards since 1950.

(C) Increasingly influential as a large sector of publishing, 20


percent of all the National and American Book awards since 1950
have gone to university-press books.

(D) Since 1950, a large and increasingly influential sector of


publishing, 20 percent of all the National and American Book
awards have gone to university-press books.

(E) Since 1950, university-press books, a large and increasingly


influential sector of publishing, won 20 percent of all the National
and American Book awards from then on.

Answers --

79). Best choice is B - Federal incentives now encourage (capital


investment) (noun) in commercial office buildings, even though
vacancy rates in existing structures are exceptionally high and there
is no demand for new construction(maintains parallelism)
A, E - incorrect - suggest structures are high and not the vacancy
rates C -awkward - passive voice
D - incorrect - it should be and instead of with, further word that
follows encourage must be a noun and not a verb.

80). B is the best choice


A, C, D - incorrect - dangling modifier ( Book awards is modifying
sector of publishing)
E - incorrect - Wrong tense..it should be present perfect tense -1950
--- till date.
Also from then on is redundant.
POSTED BY PRACHI PAREEKH AT 10:24 PM 6 COMMENTS LINKS TO THIS
POST
LABELS: MODIFIER ERROR, PARALLELISM, PASSIVE, REDUNDANCY, TENSE

Gmat Sentence Correction 77, 78

77). Oberlin College in Ohio was a renegade institution at its 1833


founding for deciding to accept both men and women as students.

(A) at its 1833 founding for deciding to accept

(B) for the decision at its 1833 founding to accept

(C) when it was founded in 1833 for its decision to accept

(D) in deciding at its founding in 1833 to accept

(E) by deciding at its founding in 1833 on the acceptance of


78). In feudal Europe, urban areas developed from clusters of houses
where peasants lived and commuted to farmlands in the
countryside, unlike homesteading policies in the American West that
required residency on the land itself in order to obtain eventual
ownership.

(A) In feudal Europe, urban areas developed from clusters of houses


where peasants lived and commuted to farmlands in the
countryside, unlike homesteading policies in the American West that

(B) In feudal Europe, urban areas developed from clusters of houses


where peasants lived and from which they commuted to farmlands
in the countryside, but in the American West homesteading policies

(C) Unlike feudal Europe where urban areas developed from clusters
of houses where peasants lived and commuted to farmlands in the
countryside, the American West's homesteading policies

(D) Unlike feudal Europe where urban areas developed from clusters
of houses where peasants lived and commuted to farmlands in the
countryside, the homesteading policies of the American West

(E) Urban areas developed from clusters of houses where peasants


lived from which they commuted to farmlands in the countryside in
feudal Europe, unlike the American West where homesteading
policies

Answers --

77). Best choice is D


A -- incorrect -- implies - as if Oberlin College was a renegade
institution specifically at its founding which was for a very short
duration.
B -- incorrect -- implies - as if there were more than one founding.
C -- incorrect -- implies - as if it was founded for the decision to
accept men and women
E -- incorrect -- wordy - "on the acceptance of."

78). The best choice is B -- Correctly comparing fuedal Europe to


American West
A, C, D -- incorrect -- comparing Europe to policies.
E -- incorrect -- awkward.
POSTED BY PRACHI PAREEKH AT 1:33 AM 0 COMMENTS LINKS TO THIS POST

LABELS: COMPARISON, WORDY

Gmat Sentence Correction 75, 76

75). It is an oversimplified view of cattle raising to say that all one


has to do with cattle is leave them alone while they feed themselves,
corral them and to drive them to market when the time is ripe.

(A) all one has to do with cattle is leave them alone while they feed
themselves, corral them, and to

(B) all one has to do with cattle is to leave them alone to feed
themselves, to corral them, and

(C) all one has to do with cattle is leave them alone while they feed
themselves and then corral them and

(D) the only thing that has to be done with cattle is leave them alone
while they feed themselves, corral them, and

(E) the only thing that has to be done with cattle is to leave them
alone while they feed themselves, to corral them, and

76). Although dozens of New York's small museums are either


devoted to local history of various ethnic groups, there are many
one-of-a-kind museums from Manhattan to the Bronx that are open
for exploration on summer weekends.

(A) Although dozens of New York's small museums are either


devoted to local history or various ethnic groups, there are

(B) Although dozens of New York's small museums are devoted to


local history or various ethnic groups,

(C) Dozens of New York's small museums are devoted to local


history or various ethnic groups, but there are

(D) Dozens of New York's small museums are devoted to local


history or various ethnic groups, and there are also

(E) Devoted to local history or various ethnic groups, dozens of New


York's small museums and also

Answers --

75). C is the best answer .


A, B -- incorrect -- parallelism "to leave them, to corral them and to
drive them" would be the correct form.
D, E -- incorrect -- use of "the only thing" is wrong as talking about
more than one action.

76). Best answer is D.


A, B -- incorrect -- use of although incorrect, it is used to show
contrast, but there is no contrast here.
C -- incorrect -- use of "but" wrong as sentence does not have a
contrast
E -- incorrect -- awkward
POSTED BY PRACHI PAREEKH AT 12:33 AM 0 COMMENTS LINKS TO THIS
POST
LABELS: PARALLELISM

SUNDAY, OCTOBER 08, 2006

Gmat Sentence Correction 73, 74

73). Building large new hospitals in the bistate area would


constitute a wasteful use of resources, on the basis of avoidance of
duplicated facilities alone.
(A) on the basis of avoidance of duplicated facilities alone

(B) on the grounds of avoiding duplicated facilities alone

(C) solely in that duplicated facilities should be avoided

(D) while the duplication of facilities should be avoided

(E) if only because the duplication of facilities should be avoided

74). Even though its per capita food supply hardly increased during
two decades, stringent rationing and planned distribution have
allowed the People’s Republic of China to ensure nutritional levels of
2,000 calories per person per day for its population.

(A) Even though its per capita food supply hardly increased during

(B) Even though its per capita food supply has hardly increased in

(C) Despite its per capita food supply hardly increasing over

(D) Despite there being hardly any increase in its per capita food
supply during

(E) Although there is hardly any increase in per capita food supply
for

Answers --

73). Passive constructions are not always wrong on GMAT


E - correct - if only because is correct idiomatic usage -- the main
clause needs to be followed logically by because hence if only
because is absolutely correct here.
A - incorrect - wordy and illogical
B,C - incorrect - duplicated facilities .... duplication should be
avoided and not the facilities...
C - incorrect - in that unidiomatic
D - incorrect - Usage of while not right

74). B is the best answer.


OE - In choice A, the simple past tense hardly increased does not
match the present perfect have allowed; consequently, it seems that
two different time periods are being discussed. In B, the best choice,
has hardly increased parallels have allowed to indicate that the
events described took place at the same time. Also in is the best word
here for making a comparison between the beginning and the end of
the twenty-year period. Choices C and D are awkward and
unidiomatic, and choice E fails to specify where there was no
increase in per capita food supply.
POSTED BY PRACHI PAREEKH AT 9:26 PM 0 COMMENTS LINKS TO THIS POST

LABELS: IDIOM, PASSIVE, TENSE

TUESDAY, SEPTEMBER 19, 2006

Gmat Sentence Correction 70, 71, 72

70). Unlike a typical automobile loan, which requires a fifteen- to


twenty-percent down payment, the lease-loan buyer is not required
to make an initial deposit on the new vehicle.

(A). the lease-loan buyer is not required to make

(B). with lease-loan buying there is no requirement of

(C). lease-loan buyers are not required to make

(D). for the lease-loan buyer there is no requirement of

(E). a lease-loan does not require the buyer to make

71). . In his research paper, Dr. Frosh, medical director of the Payne
Whitney Clinic, distinguishes mood swings. which may be violent
without their being grounded in mental disease, from genuine
manic-depressive psychosis.

(A). mood swings, which may be violent without their being


grounded in mental disease, from genuine manic-depressive
psychosis

(B). mood swings, perhaps violent without being grounded in


mental disease, and genuine manic-depressive psychosis ,

(C). between mood swings, which may be violent without being


grounded in mental disease, and genuine manic-depressive
psychosis

(D). between mood swings, perhaps violent without being grounded


in mental disease, from genuine manic-depressive psychosis

(E). genuine manic-depressive psychosis and mood swings, which


may be violent without being grounded in mental disease

72). Paleontologists believe that fragments of a primate jawbone


unearthed in Burma and estimated at 40 to 44 million years old
provide evidence of a crucial step along the evolutionary path that
led to human beings.

(A). at 40 to 44 million years old provide evidence of

(B). as being 40 to 44 million years old provides evidence of

(C). that it is 40 to 44 million years old provides evidence of what


was

(D). to be 40 to 44 million years old provide evidence of

(E). as 40 to 44 million years old provides evidence of what was

Answers -

70). Choice E, the best answer - correctly uses a parallel


construction to draw a logical comparison: Unlike a typical
automobile loan,... a lease-loan....

A - incorrect - illogically compares an automobile loan, an


inanimate thing, with a lease-loan buyer, a person.

C - incorrect - buyers makes the comparison inconsistent in number


as well as illogical.

B, D - incorrect - are syntactically and logically flawed because each


attempts to compare the noun loan and a prepositional phrase: with
lease-loan buying in B and/or the lease-loan buyer in D. These
choices are also imprecise and awkward.

71). The best choice is C - uses the idiomatically correct expression


distinguishes between x and y and because it provides a structure in
which the relative clause beginning "which may be violent" clearly
modifies "mood swings".

A, B, D, E - incorrect - use distinguishes in unidiomatic


constructions. Additionally, their in A is intrusive and unnecessary,
and the modifier of mood swings in B and D (perhaps violent) is
awkward and less clear than the more developed clause which may
be violent.

72). D, the best choice - correctly follows estimated with to be. The
other choices present structures that are not idiomatic when used in
conjunction with estimated.

A - incorrect - unidiomatic

B, C, E - incorrect - all mismatch the singular verb "provides" with


its plural subject "fragments" .Further in C and E, what was is
unnecessary and wordy. In choice C, the use of the verb phrase
estimated that it is produces an ungrammatical sentence.
POSTED BY PRACHI PAREEKH AT 12:34 AM 0 COMMENTS LINKS TO THIS
POST
LABELS: COMPARISON, IDIOM, PARALLELISM, SUBJECT VERB AGREEMENT

Gmat Sentence Correction 67, 68, 69

67). Carbon-14 dating reveals that the megalithic monuments in


Brittany are nearly 2, 000 years as old as any of their supposed
Mediterranean predecessors.

(A) as old as any of their supposed

(B) older than any of their supposed

(C) as old as their supposed

(D) older than any of their supposedly

(E) as old as their supposedly

68). The only way for growers to salvage frozen citrus is to process
them quickly into juice concentrate before they rot when warmer
weather returns.

(A) to process them quickly into juice concentrate before they rot
when warmer weather returns

(B) if they are quickly processed into juice concentrate before


warmer weather returns to rot them

(C) for them to be processed quickly into juice concentrate before the
fruit rots when warmer weather returns

(D) if the fruit is quickly processed into juice concentrate before they
rot when warmer weather returns

(E) to have it quickly processed into juice concentrate before


warmer weather returns and rots the fruit

69). Some bat caves, like honeybee hives, have residents that take on
different duties such as defending the entrance, acting as sentinels
and to sound a warning at the approach of danger, and scouting
outside the cave for new food and roosting sites.

(A) acting as sentinels and to sound

(B) acting as sentinels and sounding

(C) to act as sentinels and sound

(D) to act as sentinels and to sound

(E) to act as a sentinel sounding

Answer -

67). B is the best choice – “older than” makes the point of


comparison clear. B also uses the adjective "supposed" correctly.

A, C, E – incorrect – Illogical comparison - The expression as old as


indicates equality of age, but the sentence indicates that the
monuments by Brittany monuments predate the Mediterranean
2,000 years

D, E – incorrect – adverb “supposedly” used to modify the noun


phrase “Mediterranean predecessors”.

68). The best choice is E - has parallel infinitives and uses “to” to
refer unambiguously to citrus fruit. The cause-and-effect also
expresses the relationship between the return of warmer weather
and the rotting of the fruit. For parallelism, the linking verb “is”
should link two infinitives : The only way to salvage. . . is to process.

A – incorrect - the plural pronouns “them” and “they” do not agree


with the singular noun “citrus”.

B, C, D – incorrect - do not begin with an infinitive. and all present


pronoun errors : the plural pronouns “them” and “they”
grammatically refer to citrus or fruit and doesnot/cannot refer to
farmers.

69). B is the best choice - Because the verb phrases used to describe
the bats' duties are governed by the phrase different duties such as,
they should each be expressed in the present participial (or "-ing")
form to parallel “defending” and “scouting”. Only B uses the correct
idiom, and observes parallelism.

A, C, D, E - incorrect - all violate parallelism by employing infinitives


(to. . . ) in place of participial destinations.
POSTED BY PRACHI PAREEKH AT 12:23 AM 0 COMMENTS LINKS TO THIS
POST
LABELS: COMPARISON, MODIFIER ERROR, PARALLELISM, PRONOUN ERROR

Gmat Sentence Correction 65, 66

65). The Wallerstein study indicates that even after a decade young
men and women still experience some of the effects of a divorce
occurring when a child.

(A) occurring when a child

(B) occurring when children

(C) that occurred when a child

(D) that occurred when they were children

(E) that has occurred as each was a child

66). Since 1981, when the farm Journal began, the number of acres
overseen by professional farm-management companies have grown
from 48 million to nearly 59 million. an area that is about Colorado
's size.

(A) have grown from 48 million to nearly 59 million, an area that is


about Colorado 's size

(B) have grown from 48 million to nearly 59 million, about the size
of Colorado

(C) has grown from 48 million to nearly 59 million, an area about


the size of Colorado

(D) has grown from 48 million up to nearly 59 million, an area


about the size of Colorado 's

(E) has grown from 48 million up to nearly 59 million, about


Colorado 's size
Answers -

65). Choice D is best. The phrasing “a divorce that occurred when


they were children” correctly uses the relative clause that occurred
to modify a divorce and includes a pronoun and verb (they were)
that refer unambiguously to their antecedent. men and women.

A – incorrect - the singular child does not agree with the plural men
and women. Further this choice incorrectly introduces the when...
phrase with occurring [occurring is a progressive verb indicating
continuity in present, however we are concerned with the divorce that
occured when men or women(now) were children], thus illogically
making divorce the grammatical referent of when a child.

B – incorrect - replaces child with children but otherwise fails to


correct a 's structure and errors of logic.

C – incorrect – corrects only the error created by occurring.

E – incorrect - includes an incorrect verb tense (has occurred) and


replaces wrongly “when” with “as”. Further “each was” improperly
refers to men and women.

66). C is the best answer - An area about the size of Colorado clearly
describes a rough equivalence between the area of Colorado and the
area overseen by the companies.

A, B – incorrect - the plural verb “have” does not agree with the
singular subject “number”.

D, E – incorrect - unidiomatic; the correct expression is "from x to


y".

POSTED BY PRACHI PAREEKH AT 12:04 AM 0 COMMENTS LINKS TO THIS


POST
LABELS: IDIOM, MODIFIER ERROR, SUBJECT VERB AGREEMENT, TENSE

TUESDAY, SEPTEMBER 12, 2006

Gmat Sentence Correction 63, 64

63). Among lower- paid workers, union members are less likely than
non union members to be enrolled in lower- end insurance plans
imposing stricter limits on medical services and requiring doctors to
see more patients, and spend less time with each.

A). imposing stricter limits on medical services and requiring


doctors to see more patients, and spend

B). imposing stricter limits on medical services , requiring doctors to


see more patients, and spending

C). that impose stricter limits on medical services, require doctors to


see more patients, and spend

D). that impose stricter limits on medical services and require


doctors to see more patients, spending

E). that impose stricter limits on medical services, requiring doctors


to see more patients and spending

64). one automobile manufacturer has announced plans to increase


the average fuel efficiency of its sport utility vehicles by 25 percent
over the next five years, amounting to roughly five miles per gallon,
and representing the first significant change in the fuel efficiency of
any class of passenger vehicle in almost two decades.

A). amounting to roughly five miles per gallon, and representing

B). amounting to roughly five miles per gallon, and it would


represent

C). an increase that would amount to roughly five miles per gallon
and it would represent

D). an increase that would amount to roughly five miles per gallon
and would represent

E). which is an increase amounting to roughly five miles per gallon,


representing

Answer -

63). D is the best choice.

A,B – incorrect - insurance plans are not imposing anything

C – incorrect – Plans can’t spend time - plans that impose X, (that)


require Y and (that) spend Z

E – incorrect - plans that impose X, (that) requiring Y and (that)


spending Z. - Lacks Parallelism and plans can't spend less time

64). D is the best choice.

A,B,E -- incorrect -- What is amounting ? – an increase – Thus option


C or D

C -- incorrect -- “it” is not required.

POSTED BY PRACHI PAREEKH AT 1:31 AM 0 COMMENTS LINKS TO THIS POST

LABELS: PARALLELISM

Gmat Sentence Correction 61, 62

61). Although the first pulsar, or rapidly spinning collapsed star, to


be sighted was in the summer of 1967 by graduate student Jocelyn
Bell, it had not been announced until February, 1968.

A) Although the first pulsar, or rapidly spinning collapsed star, to be


sighted was in the summer of 1967 by graduate student Jocelyn Bell,
it had not been announced until February, 1968.

B) Although not announced until February, 1968, in the summer of


1967 graduate student Jocelyn Bell observed the first pulsar, or
rapidly spinning collapsed star, to be sighted.

C). Although observed by graduate student Jocelyn Bell in the


summer of 1967, the discovery of the first sighted pulsar, or rapidly
spinning collapsed star had not been announced before February,
1968.

D). The first pulsar, or rapidly spinning collapsed star, to be sighted


was observed in the summer of 1967 by graduate student Jocelyn
Bell, but the discovery was not announced until February, 1968.

E) The first sighted pulsar, or rapidly spinning collapsed star, was


not announced until February, 1968, while it was observed in the
summer of 1967 by graduate student Jocelyn Bell.

62). During the same period in which the Maya were developing a
heiroglphic system of writing, the Aztec people also developed a
written language, but it was not as highly sphisticated as that of the
Maya and was more pictographic in nature.

A). but it was not as highly sphisticated as that of the Maya and was

B). but it was not as highly sophisticated as the Maya, being

C). but, not as highly sophisticated as the Maya's, was

D). not as highly sphisticated as that of the Maya, however, being

E). not as highly sphisticated as the Maya's however, it was

Answer -

61). Best answer choice is D

A – incorrect – usage of past perfect tense incorrect here.

B – incorrect – dangling modifier—“Although not announced”…. “


graduate student Jocelyn Bell”

C – incorrect – dangling modifier – “observed”.. “discovery of”

E – incorrect – usage of “while” is wrong here. Further the sentence


conveys the idea that the pulsar was announced which is wrong –
sighting was announced instead.

62). A is the best choice.

B– incorrect – wrong comparison – comparing system of writing


with Maya. Further options having “being” in them must be avoided
on GMAT

C, D, E – incorrect –Subject of the sentence not clear.

E – incorrect – run-on sentence


POSTED BY PRACHI PAREEKH AT 1:23 AM 3 COMMENTS LINKS TO THIS POST

LABELS: COMPARISON, MODIFIER ERROR, TENSE

SATURDAY, AUGUST 26, 2006

Gmat Sentence Correction 60


60). A star will compress itself into a white dwarf, a neutron star, or
a black hole after it passes through a red giant stage, depending on
mass.

(A). A star will compress itself into a white dwarf, a neutron star, or
a black hole after it passes through a red giant stage, depending on
mass.

(B). After passing through a red giant stage, depending on its mass,
a star will compress itself into a white dwarf, a neutron star, or a
black hole.

(C). After passing through a red giant stage, a star's mass will
determine if it compresses itself into a white dwarf, a neutron star,
or a black hole.

(D). Mass determines whether a star, after passing through the red
giant stage, will compress itself into a white dwarf, a neutron star,
or a black hole.

(E). The mass of a star, after passing through the red giant stage,
will determine whether it compresses itself into a white dwarf, a
neutron star, or a black hole.

Answer -

60). D is the best choice -- Mass determines whether a star, after


passing through the red giant stage, will compress itself into a white
dwarf, a neutron star, or a black hole.
(Read the sentence without the words stated in orange color.)

A - incorrect -- dangling modifier...(depending on mass)

B - incorrect -- "its" refers to the red giant stage's mass.

C - incorrect -- star's mass will determine if it compress itself...again


here it refers to mass rather than star.

E - incorrect -- Mass of start after passing....it is star that is passing


and not the mass.
POSTED BY PRACHI PAREEKH AT 12:25 PM 0 COMMENTS LINKS TO THIS
POST
LABELS: MODIFIER ERROR, PRONOUN ERROR

Gmat Sentence Correction 58, 59


58). There are hopeful signs that we are shifting away from our
heavy reliance on fossil fuels: more than ten times as much energy is
generated through wind power now than it was in 1990.

A). generated through wind power now than it was

B). generated through wind power now as it was

C). generated through wind power now as was the case

D). now generated through wind power as it was

E). now generated through wind power than was the case.

59). The computer company has announced that it will purchase the
color-printing divison of a rival company for $ 950 million, which is
part of a deal that will make it the largest manufacturer in the office
color-printing market.

A). million, which is part of a deal that will make

B). million, a part of a deal that makes

C). million, part of a deal making

D). million as a part of a deal to make

E). million as part of a deal that will make

Answers -

58). C is correct answer - correct idiom “as X as Y”

A,B,D – incorrect – no clear referent for “it”

E – incorrect – “than” is used incorrectly in this option.

59). E is the best choice.

B,C,D – incorrect –wrong tense usage -- present tense is used instead


of future tense

A – incorrect – “which” –referrent not clear..

Note – Always remember a participle phrase as in C here, is used for


an event in the past or which is going on at present but never for the
future – the reason why again C is wrong. C is basically changing
the meaning of the sentence.
POSTED BY PRACHI PAREEKH AT 12:17 PM 0 COMMENTS LINKS TO THIS
POST
LABELS: IDIOM, PRONOUN ERROR, TENSE

Gmat Sentence Correction 56, 57

56). Some patients who do not respond therapies of depression may


simply have received inadequate treatment, having, for example
been prescribed a drug as a dosage too low to be effective or having
been taken off a drug too soon.

(A). having, for example been prescribed a drug as a dosage too low
to be effective or having been

(B). having, for example, a drug prescription that was ineffective


because the dosage was too low, or being

(C). as, for example, having too low of a dosage of prescribed drug
for it to be effective, or being

(D). when they have, for example, been prescribed too low a drug
dosage for it to be effective, or were

(E). for example, when they have a drug prescription with a dosage
too low to be effective, or been

57). Officials at the United States Mint believe that the Sacagawea
dollar coin will be used more as a substitute for four quarters rather
than for the dollar bill because its weight, only 8.1 grams, is far less
than four quarters, which weigh 5.67 grams each.

(A). more as a substitute for four quarters rather than for the dollar
bill because its weight, only 8.1 grams, is far less than

(B). more as a substitute for four quarters than the dollar bill
because it weighs only 8.1 grams, far lighter than

(C). as a substitute for four quarters more than for a dollar bill
because it weighs only 8.1 grams, far less than

(D). as a substitute for four quarters more than the dollar bill
because its weight of only 8.1 grams is far lighter than it is for
(E). as a substitute more for four quarters rather than for the dollar
bill because its weight, only 8.1 grams, is far less than it is for

Answers -

56). A is the best choice. “having….been” is correct usage as it


maintains parallelism.

B – incorrect – idea conveyed by this option is that a drug


prescription was ineffective which cannot be true.

C – incorrect -- parallelism is violated here – having … being

D – incorrect -- wrong tense and violating parallelism

E – incorrect -- violating parallelism

57). C is the best choice.

A,D,E – incorrect – violating parallelism – it should be “weighs” and


not “weights”

B – incorrect – incorrectly comparing “as a substitute” with “four


quarters”

POSTED BY PRACHI PAREEKH AT 12:06 PM 3 COMMENTS LINKS TO THIS


POST
LABELS: COMPARISON, PARALLELISM, TENSE

Gmat Sentence Correction 54, 55

54). A new phenomena, which is visible at Managua's major


intersections, are waves of vendors and beggars, which include
many children and mob cars at the stoplights.

(A) A new phenomena, which is visible at Managua's major


intersections, are waves of vendors and beggars, which include
many children and

(B) Visible at Managua's major intersections are waves of vendors


and beggars with many children, new phenomena that

(C) A new phenomenon visible at Managua's major intersections is


waves of vendors and beggars, many of them children, who
(D) Phenomenally new waves of vendors, beggars, and many
children are visible at Managua's major intersections, which

(E) A wave of vendors and beggars, many of whom are children, are
visible at Managua's major intersections, where they are a new
phenomenon and

55). To meet the rapidly rising market demand for fish and seafood,
suppliers are growing fish twice as fast as their natural growth rate,
cutting their feed allotment by nearly half and raising them on
special diets.

(A). their natural growth rate, cutting their feed allotment

(B). their natural growth rate, their feed allotment cut

(C). growing them naturally, cutting their feed allotment

(D). they grow naturally, cutting their feed allotment

(E). they grow naturally, with their feed allotment cut

Answers -

54). C is the best choice.

A,E - incorrect -- Subject-Verb Agreemant

B - incorrect -- phenomenon cannot rob cars.

D - incorrect -- very awkward , changes the meaning of the sentence

55). D is the correct answer - parallelism is maintained


-"growing....cutting...raising...". Further antecedent of “they” is
clear i.e “the fish”

C - incorrect -- after “as fast as” we need a clause.

A, B - incorrect -- wrong comparison -- cannot compare verb with


noun --- growing fish (verb) with growth rate (noun).

B, E - incorrect -- violating parallelism.


POSTED BY PRACHI PAREEKH AT 11:48 AM 0 COMMENTS LINKS TO THIS
POST
LABELS: COMPARISON, PARALLELISM, SUBJECT VERB AGREEMENT

Gmat Sentence Correction 52, 53

52). Byron possessed powers of observation that would have made


him a great anthropologist and that makes his letters as a group the
rival of the best novels of the time.

(A) makes his letters as a group the rival of

(B) makes his letters as a group one to rival

(C) makes his letters a group rivaling

(D) make his letters as a group the rival of

(E) make his letters a group which is the rival of

53). The expected rise in the price of oil could be a serious impact to
industrialized nations and severely diminish the possibility to have
an economy free of inflation.

(A) be a serious impact to industrialized nations and severely


diminish the possibility to have

(B) seriously impact on industrialized nations and severely impede


the possibility to have

(C) seriously impact on industrialized nations and severely impede


the possibility of having

(D) have a serious impact on industrialized nations and severely


impede the possibility to have

(E) have a serious impact on industrialized nations and severely


diminish the possibility of having

Answers -

52). Correct answer - D

A,B,C - incorrect - subject verb agreement - powers....make

E - wordy

53). E is the best choice.


A,B,C - incorrect -- Always remember in GMAT --impact should
remain a noun; a proposal can have an impact.(The only thing that
can be impacted is a wisdom tooth).

D - incorrect - "possibility of" is correct idiom. Further "diminish"


instead of "impede" is more appropriate to use here.

50). In large doses, analgesics that work in the brain as antagonists


to certain chemicals have caused psychological disturbances in
patients, which may limit their potential to relieve severe pain.

(A) which may limit their potential to relieve

(B) which may limit their potential for relieving

(C) which may limit such analgesics’ potential to relieve

(D) an effect that may limit their potential to relieve

(E) an effect that may limit the potential of such analgesics for
relieving

51). The man was always aware, sometimes proudly and sometimes
resentfully, that he was a small-town Midwesterner who was thrust
into a world that was dominated by wealthier, better-educated, and
more polished people than him.

(A) who was thrust into a world that was dominated by wealthier,
better-educated, and more polished people than him

(B) who had been thrust into a world that was dominated by more
wealthy, educated, and polished people than him

(C) who had been thrust into a world dominated by wealthier,


better-educated, and people more polished than he was

(D) thrust into a world dominated by more wealthy, educated, and


polished people than him

(E) thrust into a world dominated by wealthier, better-educated,


and more polished people than he

Answers -
50). E is the best choice

A,B,C - incorrect - use of "which" ambiguous - referring to 'patients'


or 'analgesics' or..

D - incorrect - "their" -ambiguous - referring to 'patients' or


'analgesics' or effects of the analgesics...

51). E is the correct answer.

A,B,D- incorrect - Subject form of pronoun always comes after ‘than’


or ‘as’. (Pronoun errors)

C - incorrect - ".. than he was" - was needs to be ellipsed as I


mentioned for Do - auxiliary verb in my GMAT Grammar blog.
POSTED BY PRACHI PAREEKH AT 11:03 AM 0 COMMENTS LINKS TO THIS
POST
LABELS: PRONOUN ERROR

SATURDAY, AUGUST 12, 2006

Gmat Sentence Correction 49

49). With a total population of less than two hundred and fewer
breeding females than ever before, the American crocodile seemed a
decade ago to be in danger of disappearing.

(A) of less than two hundred and fewer

(B) lower than two hundred and less

(C) lesser than two hundred and fewer

(D) fewer than two hundred and less

(E) of fewer than two hundred and of fewer

Answer -

49). A is the right answer.

D, E - incorrect -- use "fewer" only when referring to countable


nouns. Population is very much like "mass" here and so less/lesser
should be used.
B,C - incorrect -- we require 'of ' after population and before less, and
"fewer" for refering to the number of breeding females.
POSTED BY PRACHI PAREEKH AT 9:53 AM 0 COMMENTS LINKS TO THIS POST

LABELS: LESS VS FEWER

MONDAY, JULY 31, 2006

Gmat Sentence Correction 47, 48

47). Most state constitutions now mandate that the state budget be
balanced each year.

(A) mandate that the state budget be balanced

(B) mandate the state budget to be balanced

(C) mandate that the state budget will be balanced

(D) have a mandate for a balanced state budget

(E) have a mandate to balance the state budget

48). Two new studies indicate that many people become obese more
due to the fact that their bodies burn calories too slowly than
overeating.

(A) due to the fact that their bodies burn calories too slowly than
overeating

(B) due to their bodies burning calories too slowly than to eating too
much

(C) because their bodies bum calories too slowly than that they are
overeaters

(D) because their bodies bum calories too slowly than because they
eat too much

(E) because of their bodies burning calories too slowly than because
of their eating too much

Answers -
47). A is the correct answer - When mandate is used as a verb to
mean "make it mandatory,' it must be followed by that and a verb in
the
subjunctive mood, as in A, the best answer: mandate that x be
balanced.

B - incorrect -- uses the ungrammatical mandate x to be balanced.

C - incorrect -- inappropriately uses the future indicative, will be,


rather than the subjunctive.

D, E - incorrect -- use wordy and imprecise expressions in place of


the verb mandate: neither have a mandate for a balanced... budget
nor have a mandate to balance the ... budget makes clear that the
requirement is made by the constitution. It is also unclear in D
whether each year refers to the mandating or the balancing.

48). D is the best choice -- The members of a comparison (more X


than Y) should be expressed in parallel form. This choice correctly
uses parallel clauses introduced by because. The clauses themselves
are clear and direct.

E - incorrect -- uses parallel forms, but the convoluted structures are


awkward and wordy. Furthermore, the word "bodies" would need
an apostrophe (bodies') since it is the logical subject of the gerund
burning (that is, it answers the question, "Whose burning?").

A, B, C - incorrect -- do not use parallel forms for the two members of


the comparison. In addition, A and B use "due to" unidiomatically to
mean "because"; properly used, "due to" is synonymous with
"attributable to".
POSTED BY PRACHI PAREEKH AT 11:12 PM 0 COMMENTS LINKS TO THIS
POST
LABELS: COMPARISON, PARALLELISM, SUBJUNCTIVE, WORDY

Gmat Sentence Correction 45 , 46

45). The Coast guard is conducting tests to see whether pigeons can
be trained to help find survivors of wrecks at sea.

(A) to see whether pigeons can be trained to help find

(B) to see whether pigeons can be trained as help to find

(C) to see if pigeons can be trained for helping to find


(D) that see if pigeons are able to be trained in helping to find.

(E) that see whether pigeons are able to be trained for help in
finding

46). The first decision for most tenants living in a building


undergoing being converted to cooperative ownership
is if to sign a no-buy pledge with the other tenants.

(A) being converted to cooperative ownership is if to sign

(B) being converted to cooperative ownership is whether they


should be signing

(C) being converted to cooperative ownership is whether or not they


sign

(D) conversion to cooperative ownership is if to sign

(E) conversion to cooperative ownership is whether to sign

Answers -

45). A is the correct answer -- This choice clearly states both the
purpose of the test and the purpose of the training.

B - incorrect -- "As help to find" is unidiomatic.

C - incorrect -- "For helping to find" is not the correct idiom;


"whether" is preferred to "if" when there are only two alternatives.

D - incorrect -- "That see " is ungrammatical; "whether" is preferred


to "if" when there are only two alternatives; "are able to be" should
be replaced by the concise "can"; "in helping to find" should be " to
help find".

E - incorrect -- "For helping in finding" is incorrect idiom; "are able


to be" should be replaced by the concise "can".

46). E is the best choice -- idiomatically completes "whether" with an


infinitive, "to sign", that functions as a noun equivalent of decision.
It also uses the noun conversion, which grammatically completes
the phrase begun by "undergoing".

A, B, C - incorrect -- the phrase "being converted" is awkward and


redundant, since the sense of process indicated by
"being" has already been conveyed by "undergoing".

A, D - incorrect -- can be faulted for saying "if" rather than


"whether", since the sentence poses alternative possibilities, to sign
or not to sign.
POSTED BY PRACHI PAREEKH AT 10:44 PM 0 COMMENTS LINKS TO THIS
POST
LABELS: IDIOM, REDUNDANCY, WHETHER VS IF

FRIDAY, JULY 28, 2006

Gmat Sentence Correction 43, 44

43). The Diary of Anne Frank tells the true story of a young girl and
her family that were hidden during the Nazi occupation of the
Netherlands by a gentile Dutch couple, though they were eventually
discovered.

A). that were hidden during the Nazi occupation of the Netherlands
by a gentile Dutch couple, though they were eventually discovered

B). that were hidden by a gentile Dutch couple during the Nazi
occupation of the Netherlands, though they were eventually
discovered

C). whom a gentile Dutch couple hid during the Nazi occupation of
the Netherlands but were eventually discovered

D). who were hidden by a gentile Dutch couple during the Nazi
occupation of the Netherlands but were eventually discovered

E). who were hidden by a gentile Dutch couple during the Nazi
occupation of the Netherlands even though they were eventually
discovered

44). Despite the sequel's poor critical reception, most people seeing
it find the acting and cinematography at least as good or even better
than the original.

A). most people seeing it find the acting and cinematography at least
as good or even better than the original
B). most people seeing it find the acting and cinematography at least
as good or even better than the original's

C). most people who see the film find the acting and cinematography
at least as good as or even better than those in the original

D). most people who see it find the acting and cinematography at
least as good or even better than those in the original

E). most people seeing the film find the acting and cinematography
at least as good as or even better than those of the original's

Answers -

43). The correct answer is D -- It correctly uses the pronoun "who" to


refer to a girl and her family. Additionally, the phrase "by a gentile
Dutch couple" is placed immediately after "who were hidden" to
clarify the meaning. Finally, the ambiguous pronoun issue is
avoided altogether.

A - incorrect -- The original sentence uses the relative pronoun "that"


where "who" is preferred because the antecedent is a group of
people. Also, the prepositional phrase "by a gentile Dutch couple" is
placed in such a way as to suggest that the occupation was carried
out by the couple. Finally, the pronoun "they" is ambiguous – it
could refer to the family or to the couple.

B - incorrect -- This choice incorrectly uses the relative pronoun


"that" to refer to a group of people. Additionally, the pronoun "they"
is ambiguous - it could refer to the family or to the couple.

C - incorrect -- The use of the object pronoun "whom" makes "a girl
and her family" the object of the clause "a gentile Dutch couple hid;"
however "a girl and her family" are the subject of the next clause
"were eventually discovered." This is a mismatch.

E - incorrect -- The pronoun "they" is ambiguous - it could refer to


the family or to the couple.

44). The correct answer is C -- This choice correctly replaces "it"


with "film", adds the missing "as", and makes clear that the acting
and cinematography in the sequel are compared to the acting and
cinematography in the original.

A - incorrect -- First, the pronoun "it" is used to refer to the film, but
the film has not been mentioned in the sentence. Instead "the
sequel's poor critical reception" has been mentioned. This is not the
same as the film, so the pronoun "it" has no grammatical antecedent
and must be changed.
Second, the phrase "at least as good or even better than" is
incomplete: there should be another "as" after "good".
Third, the cinematography and acting are being compared to the
"original". What is meant here is that the cinematography and
acting in the sequel are as good as the cinematography and acting in
the original. This must be changed.

B - incorrect -- The pronoun "it" has no grammatical antecedent and


the phrase "at least as good or even better" is missing the second
"as" after "good."

D - incorrect -- The pronoun "it" has no grammatical antecedent and


the phrase "at least as good or even better" is missing the second
"as" after "good."

E - incorrect -- This choice introduces the possessive redundancy


"those of the original's." One could say either "those of the original"
or "the original's", but using both is redundant.
POSTED BY PRACHI PAREEKH AT 1:54 AM 3 COMMENTS LINKS TO THIS POST

LABELS: PRONOUN ERROR, REDUNDANCY

SUNDAY, JULY 23, 2006

Gmat Sentence Correction 41, 42

41). Starting at age four, Mozart's father began taking him on tours
of the capitals of Europe, in order to demonstrate his musical
talents.

(A). Mozart's father began taking him on tours of the capitals of


Europe, in order to demonstrate his musical talents

(B). Mozart's father began taking the boy on tours of the capitals of
Europe, to demonstrate his musical talents

(C). Mozart began accompanying his father on tours of the capitals


of Europe, to demonstrate his own musical talents

(D). Mozart was accompanying his father on tours of Europe's


capitals, to demonstrate his musical talents
(E). Mozart's father began taking him on tours of the capitals of
Europe, to demonstrate the boy's musical talents

42). According to the professor's philosophy, the antidote to envy is


one's own work, always one's own work: not thinking about it, not
assessing it, but simply doing it.

(A) one's own work, always one's own work: not thinking about it,
not assessing it, but simply doing it

(B) always work; because you don't think about it or assess it, you
just do it

(C) always one's own work: not thinking about or assessing it, but
simply to do it

(D) not to think or assess, but doing one's own work

(E) neither to think about one's own work nor to assess it, it is
always simply doing it

Answers -

41). C is the best choice -- "Mozart" is correctly placed as the subject


of the opening modifier "starting at age four." Additionally, the
phrase "his own" clarifies that the musical talents in question are
those of the subject, Mozart.

A - incorrect -- The original sentence begins with a modifier


("starting at age four") that describes Mozart. Yet, the subject of the
main clause is Mozart's father. We need to find a choice that make
Mozart himself the subject. Moreover, the pronoun "him" has no
grammatical antecedent, since it is meant to refer to Mozart despite
the fact that Mozart is not actually present in the sentence (we have
"Mozart's father" instead). Finally, the pronoun "his" is somewhat
ambiguous: does it refer to Mozart (who, again, is not in the
sentence) or to his father?

B -- incorrect -- "Mozart's father" serves illogically as the subject of


the opening modifier "starting at age four." Additionally, the
pronoun "his" is ambiguous in that it could refer to the boy or the
father.

D -- incorrect -- This choice incorrectly uses the verb "was," which


does not match the ongoing nature of the modifier "starting at age
four." Additionally, the pronoun "his" is ambiguous in that it could
refer to Mozart or his father.

E -- incorrect -- "Mozart's father" serves illogically as the subject of


the opening modifier "starting at age four." Additionally, the
pronoun "him" has no grammatical antecedent, since it is meant to
refer to Mozart despite the fact that Mozart is not actually present in
the sentence (we have "Mozart's father" instead).

42). The correct answer is A - maintains parallelism - thinking,


assessing, doing

D, E - incorrect - violates parallelism -- think, assess, doing are not


parallel

C - incorrect - violates parallelism -- thinking, assessing and to do it


are not parallel.

B - incorrect -- the antidote to envy is to work always -- changes the


meaning of the original sentence.
POSTED BY PRACHI PAREEKH AT 11:40 PM 0 COMMENTS LINKS TO THIS
POST
LABELS: MODIFIER ERROR, PARALLELISM, PRONOUN ERROR

Gmat Sentence Correction 38, 39, 40.

38). The Parthenon was a church from 1204 until 1456, when Athens
was taken by General Mohammed the Conqueror, the Turkish
sultan, who established a mosque in the building and used the
Acropolis as a fortress.

(A) who established a mosque in the building and used the Acropolis
as

(B) who, establishing a mosque in the building, used the Acropolis


like

(C) who, when he had established a mosque in the building, used the
Acropolis like

(D) who had established a mosque in the building, using the


Acropolis to be

(E) establishing a mosque in the building and using the Acropolis as


39). The concept of the grand jury dates from the twelfth -century,
when Henry II of England ordered panels of common citizens should
prepare lists of who were their communities' suspected criminals.

(A) should prepare lists of who were their communities' suspected


criminals

(B) would do the preparation of lists of their communities' suspected


criminals

(C) preparing lists of suspected criminals in their communities

(D) the preparing of a list of suspected criminals in their


communities

(E) to prepare lists of suspected criminals in their communities

40). Chinese, the most ancient of living writing systems, consists of


tens of thousands of ideographic characters, each character
miniature calligraphic composition inside its own square frame.

(A) each character a miniature calligraphic composition inside its

(B) all the characters a miniature calligraphic ; composition inside


their

(C) all the characters a miniature calligraphic composition inside its

(D) every character a miniature calligraphic composition inside


their

(E) each character a miniature calligraphic composition inside their

Answers -

38). The best answer is A -- correctly supplies the past tense verbs
"established" and "used" to describe two actions performed in 1456;
also, it idiomatically employs the phrase "used the Acropolis as a
fortress", in which used as means "employed in the capacity of."

B, C -- incorrect -- incorrectly replaces "as" with "like". Furthermore,


in
C, "when he had established a mosque" distorts the intended
meaning by stating that the first action was completed before the
second was begun.

D -- incorrect -- "had established... using" states that Mohammed


had already performed the actions before capturing Athens; In
addition D includes the unidiomatic construction "using x to be y."

E -- incorrect -- "establishing" and "using" modify Athens, thus


producing an absurd statement.

39). Choice E is best -- the infinitive "to prepare" follows the verb
"ordered", producing the grammatical and idiomatic sequence - x
ordered y to do z.

A, B -- incorrect -- "should prepare" in A and "would do" in B


produce ungrammatical sequences - x ordered y should/ would do z.

C -- incorrect -- "preparing . . . communities functions" as a


participial
phrase is modifying citizens rather than as a verb phrase describing
what the citizens were ordered to do.

D -- incorrect -- the construction "ordered panels of common citizens


the preparing" is unidiomatic.

40). Choice A is best -- the appositive terms "character" and


"composition", both singular, agree in number; both also agree with
the singular possessive pronoun "its".

B, C, D, E -- incorrect -- three-way agreement in number stated in


choice A is violated.
POSTED BY PRACHI PAREEKH AT 10:44 AM 2 COMMENTS LINKS TO THIS
POST
LABELS: IDIOM, LIKE VS AS, MODIFIER ERROR, SUBJECT VERB
AGREEMENT, TENSE

Gmat Sentence Correction 36, 37

36). In contrast to large steel plants that take iron ore through all
the steps needed to produce several different kinds of steel,
processing steel scrap into a specialized group of products has
enabled small mills to put capital into new technology and remain
economically viable.
(A) processing steel scrap into a specialized group of products has
enabled small mills to put capital into new technology and remain

(B) processing steel scrap into a specialized group of products has


enabled small mills to put capital into new technology, remaining

(C) the processing of steel scrap into a specialized group of products


has enabled small mills to put capital into new technology,
remaining

(D) small mills, by processing steel scrap into a specialized group of


products, have been able to put capital into new technology and
remain

(E) small mills, by processing steel scrap into a specialized group of


products, have been able to put capital into new technology and
remained

37). Any medical test will sometimes fail to detect a condition when
it is present and indicate that there is one when it is not.

(A) a condition when it is present and indicate that there is one

(B) when a condition is present and indicate that there is one

(C) a condition when it is present and indicate that it is present

(D) when a condition is present and indicate its presence

(E) the presence of a condition when it is there and indicate its


presence

Answers -

36). D is the correct answer -- uses parallel verb forms to complete


the construction "have been able to put... and remain". The logical
comparison here is between large steel plants and small mills.

A, B,C -- incorrect -- illogically contrast large steel plants with [the]


processing [of] steel scrap. Further, in choices B and C remaining is
not parallel with put; consequently, it is not clear exactly what is
remaining economically viable.

E -- incorrect -- remained is not parallel with put.


37). Correct choice is C -- produces a sentence in which pronoun "it"
refers clearly and logically to the noun "condition".

A, B -- incorrect -- the phrase "indicate that there . is one" does not


grammatically fit with "when it is not" because it has no referent.

B, D -- incorrect -- are imprecise in saying that a test will fail to


detect
when a condition is present, since the issue is the presence and not
the timing of the condition. Further, "its" presence in D leaves the
"it" in "when it is not" without a logical referent: it must refer to
condition, not presence.

E -- incorrect -- repeats the error stated above ; also, "the presence ...
when it is there" is imprecise and redundant.
34). Though viewed from a distance, Saturn's main rings may
appear to be smooth and continuous, they are in fact composed of
thousands of separate icy ringlets when viewed up close.

A). Though viewed from a distance, Saturn's main rings may appear
to be smooth and continuous, they are in fact composed of
thousands of separate icy ringlets when viewed up close.

B). Though Saturn's main rings may appear smooth and continuous
when viewed from a distance, they are in fact composed of
thousands of separate icy ringlets when viewed up close.

C). Saturn's main rings, when viewed from a distance, may appear
to be smooth and continuous, though when viewed up close they are
in fact composed of thousands of separate icy ringlets.

D). When viewed from a distance, Saturn's main rings may appear
smooth and continuous, but closer viewing reveals them to be
composed of thousands of separate icy ringlets.

E). Though composed of thousands of separate icy ringlets if viewed


up close, the main rings of Saturn may appear smooth and
continuous when they are viewed from a distance.

35). The ancient Inca city of Macchu Picchu, perched on a ridge in


the Andes Mountains of Peru, had been built at a high enough
altitude that it often makes modern-day tourists from lower
elevations sick with oxygen deprivation.

A). had been built at a high enough altitude that it often makes
modern-day tourists from lower elevations sick with oxygen
deprivation
B). had been built at so high of an altitude that it often makes
modern-day tourists from lower elevations sick from oxygen
deprivation

C). was built at a high enough altitude that modern day tourists
from lower elevations often become sick with oxygen deprivation
when visiting the city

D). was built at such a high altitude that modern-day tourists from
lower elevations often become sick from oxygen deprivation when
visiting the city

E). was built at so high of an altitude that it often makes modern-


day tourists from lower elevations sick from oxygen deprivation

Answer -

34). D is the correct answer - This sentence checks the clarity of


meaning and modifiers. This choice replaces "though" with "when"
and shortens "appear to be" to "appear." Further, its use of the
phrase "closer viewing reveals" clearly indicates that the close
viewing only reveals (not causes) the composition of the rings.

A - incorrect - The original sentence introduces the main clause with


"though viewed from a distance", which sets up the expectation of a
contradiction that never materializes. For example, "Though sleepy,
the child stayed awake" is correct, whereas "Though sleepy, the
child may have eaten soup" is not.

Also, "when viewed up close" is placed in such a way as to illogically


suggest that the rings are composed of icy ringlets as a result of
being viewed up close.

Finally, "appear to be" is redundant.

B - This choice incorrectly introduces the main clause with "though."


Additionally, the placement of "when viewed up close" illogically
suggests that the rings are composed of icy ringlets as a result of
being viewed up close.

C - This choice incorrectly uses the redundant phrase "appears to


be." Additionally, the use and placement of the words "when viewed
up close, they are . . ." illogically suggests that the rings are
composed of icy ringlets as a result of being viewed up close.

E - This choice incorrectly introduces the main clause with "though."


Additionally, the placement of "if viewed up close" illogically
suggests that the rings are composed of icy ringlets as a result of
being viewed up close.

35). D is the correct answer - The simple past verb "was" correctly
replaces the past perfect verb "had been." The phrase "such a high
altitude" replaces "high enough altitude." The sentence is rewritten
to avoid ambiguity by removing the pronoun "it." Finally, this
choice uses the phrase "sick from" in place of the unidiomatic "sick
with."

A - incorrect - First, the past perfect "had been" is unnecessary here


because there is only one past event. Second, "high enough" implies
that the oxygen deprivation was a goal of the Incas. Third, "it" in
this context is ambiguous: is it the altitude or the city that makes
tourists sick? Finally, "sick with" is incorrect. It should be "sick
from."

B - incorrect - This choice incorrectly uses the past perfect "had


been." Additionally, the pronoun "it" is ambiguous in this context
and the phrase "so high of an altitude" is awkward.

C - This choice incorrectly uses the phrases "high enough altitude"


and "sick with."

E - This choice incorrectly uses the awkward phrase "so high of an


altitude." Additionally, the pronoun "it" in this context is
ambiguous.

POSTED BY PRACHI PAREEKH AT 11:07 PM 1 COMMENTS LINKS TO THIS


POST
LABELS: MODIFIER ERROR, PRONOUN ERROR, REDUNDANCY, TENSE

TUESDAY, JULY 04, 2006

Gmat Sentence Correction 32 , 33

32). . Scientists have observed large concentrations of heavy-metal


deposits in the upper twenty centimeters of Baltic Sea sediments,
which are consistent with the growth of industrial activity there.

(A) Baltic Sea sediments, which are consistent with the growth of
industrial activity there

(B) Baltic Sea sediments, where the growth of industrial activity is


consistent with these findings

(C) Baltic Sea sediments, findings consistent with its growth of


industrial activity

(D) sediments from the Baltic Sea, findings consistent with the
growth of industrial activity in the area

(E) sediments from the Baltic Sea, consistent with the growth of
industrial activity there

33). For members of the seventeenth-century Ashanti nation in


Africa, animal-hide shields with wooden frames were essential items
of military equipment, a method to protect warriors against enemy
arrows and
spears.

(A) a method to protect

(B) as a method protecting

(C) protecting

(D) as a protection of

(E) to protect

Answers -

32). D is the best choice - the phrase sediments from the Baltic Sea
tells where the sediments originate, findings provides a noun for
consistent to modify, and in the area clearly identifies where the
industrial activity is growing.

A, B - incorrect - ambiguity - the words "which" and "where" appear


to refer to sediments

E - incorrect - it is not clear what consistent describes.

A, C, E - incorrect - there is no logical place to which "there" or "its"


could refer.

33). Choice C is best - the participle "protecting" begins a phrase that


explains what the shields did.

A , B - incorrect - awkwardly use the singular word "method" to


refer to items of military equipment rather than to the use of such
items. Also, a method of protecting would be more idiomatic than a
method to protect in A or a method protecting in B.

B , D - "as" is incorrect; also, a protection in D has no noun for which


it can logically substitute.

E - is incomplete; used to protect would have been acceptable

POSTED BY PRACHI PAREEKH AT 4:24 AM 0 COMMENTS LINKS TO THIS POST

LABELS: PRONOUN ERROR

MONDAY, JULY 03, 2006

Gmat sentence correction 30 , 31

30). Famed for his masterful use of irony, many of Guy de


Maupassant's short stories have become classics due to the author
slowly revealing at the end of each piece a tragic twist of fate.

A). Famed for his masterful use of irony, many of Guy de


Maupassant's short stories have become classics due to the author
slowly revealing at the end of each piece a tragic twist of fate.

B). Many of Guy de Maupassant's short stories have become classics


because of how he famously and masterfully uses irony, evident in
the slow revelation of a tragic twist of fate at the end of each piece.

C).Famed for using irony in a masterful way, many of Guy de


Maupassant's short stories have become classics because of the
author slowly revealing a tragic twist of fate at the end of each
piece.

D). Many of Guy de Maupassant's short stories have become classics


because of the author's famed and masterful use of irony, evidenced
in the slow revelation of a tragic twist of fate at the end of each
piece.

E). Many of Guy de Maupassant's short stories have become classics


because he slowly revealed a tragic twist of fate at the end of each
piece, demonstrating his famed and masterful use of irony.

31). Teachers in this country have generally been trained either to


approach mathematics like a creative activity or that they should
force students to memorize rules and principles without truly
understanding how to apply them.

A). to approach mathematics like a creative activity or that they


should force students to memorize rules and principles

B). to approach mathematics like a creative activity or to force


students to memorize rules and principles

C). to approach mathematics as a creative activity or to force


students to memorize rules and principles

D). that they should approach mathematics as a creative activity or


to force students to memorize rules and principles

E). that they should approach mathematics like a creative activity


or that they should force students to memorize rules and principles

Answers -

30). D is the correct answer

A - Incorrect - sentence begins with the modifier "Famed for his


masterful use of irony," which requires a person as its subject.
However, in the sentence, "many of Guy de Maupassant's short
stories" is the subject. Moreover, the phrase "due to the author
slowly revealing" is awkward.

B - incorrect - The pronoun "he" must have a person as its


antecedent, yet there is no person in the sentence. Remember that
"he" cannot refer to "Guy de Maupassant" here, since the name is
part of a possessive phrase: "Guy de Maupassant's short stories".
The author himself is not grammatically present in the sentence.

C - incorrect - The opening modifier "famed for using irony in a


masterful way" incorrectly modifies "short stories" instead of Guy
de Maupassant himself. It also contains the awkward phrase
"because of the author slowly revealing."

E - incorrect - This choice incorrectly uses the pronoun "he" without


a grammatical antecedent in the sentence.

31). C is the correct choice - The construction "either X or Y" requires


parallelism between X and Y. In choice C, both X and Y are parallel
infinitive phrases ("to approach . . ." and "to force . . .").

A - incorrect - incorrectly pairs an infinitive ("to approach") with a


clause ("that they should...") in the construction "either X or Y."
Moreover, the use of "like" in the phrase "to approach mathematics
like a creative activity" is incorrect. :"As" should be used instead.

B - While this choice does contain proper parallel structure, it


incorrectly uses "like" instead of "as" in the phrase "to approach
mathematics like a creative activity".

D - This choice incorrectly pairs a clause ("that they should...") with


an infinitive ("to approach") in the construction "either X or Y."

E - While this choice does create a parallel construction, it


awkwardly begins the parallel elements with the words "that they"
instead of the infinitive "to." Moreover, this choice incorrectly uses
"like" instead of "as" in the phrase "to approach mathematics like a
creative activity".
POSTED BY PRACHI PAREEKH AT 6:53 AM 1 COMMENTS LINKS TO THIS POST

LABELS: MODIFIER ERROR, PARALLELISM, PRONOUN ERROR

SUNDAY, JULY 02, 2006

Gmat sentence correction 26 , 27 , 28 , 29

26). A study commissioned by the Department of Agriculture showed


that if calves exercise and associated with other calves, they will
require less medication and gain weight quicker than do those
raised in confinement.

(A) associated with other calves, they will require less medication
and gain weight quicker than do

(B) associated with other calves, they require less medication and
gain weight quicker than

(C) associate with other calves, they required less medication and
will gain weight quicker than do

(D) associate with other calves, they have required less medication
and will gain weight more quickly than do

(E) associate with other calves, they require less medication and
gain weight more quickly than

27). Displays of the aurora borealis, or "northern lights," can heat


the atmosphere over the arctic enough to affect the trajectories of
ballistic missiles, induce electric currents that can cause blackouts
in some areas and corrosion in north-south pipelines.

(A) to affect the trajectories of ballistic missiles, induce

(B) that the trajectories of ballistic missiles are affected, induce

(C) that it affects the trajectories of ballistic missiles, induces

(D) that the trajectories of ballistic missiles are affected and induces

(E) to affect the trajectories of ballistic missiles and induce

28). The golden crab of the Gulf of Mexico has not been fished
commercially in great numbers, primarily on account of living at
great depths-- 2,500 to 3,000 feet down.

(A) on account of living

(B) on account of their living

(C) because it lives

(D) because of living

(E) being they live

29). Delighted by the reported earnings for the first quarter of the
fiscal year, it was decided by the company manager to give her staff
a raise.

(A) it was decided by the company manager to give her staff a raise

(B) the decision of the company manager was to give her staff a
raise

(C) the company manager decided to give her staff a raise

(D) the staff was given a raise by the company manager

(E) a raise was given to the staff by the company manager

Answers -

26). Choice E, the best answer, uses the adverbial phrase "more
quickly than" to modify the verb phrase "gain weight".
In A, B, and C - "quicker than" is incorrect because an adjective
should not be used to modify a verb phrase.

A and B - incorrectly compound that present tense verb with a past


tense verb, "associated".

C and D - correctly use "associate", but C follows with the past tense
"required" and D with the present perfect "have required". Both C
and D incorrectly conclude with the future tense "will gain".

27). E is the best choice.

The use of the phrasing "can heat... enough to affect" in A and E is


more idiomatic than the use of the subordinate clause beginning
with "that" in B, C, and D.

B - produces an illogical and ungrammatical statement by making


"induce" parallel with the verb "heat" rather than with the
appropriate form of the verb affect

C - lacks agreement in using the singular pronoun "it" to refer to the


plural noun "displays"

D - is faulty because "induces" cannot fit grammatically with any


noun in the sentence.

A - incorrectly separates the two infinitives "to affect" and "to


induce" with a comma when it should compound them with "and",
as does E, the best choice.

28). C is the best choice.

A, B, D - the phrases "on account of" and "because of" are


unidiomatic; "because", which appears in C and E, is preferable here
since "because" can introduce a complete subordinate clause
explaining the reason why the golden crab has not been fished
extensively.

B , E - also produce agreement errors by using the plural pronouns


"their" and "they" to refer to the singular noun "crab".

D - like A, fails to provide a noun or pronoun to perform the action of


living.

C - which uses "because" and "it" as the singular subject of a clause,


is the best choice.
29). C is the best answer. Grammatically, the participial phrase
beginning delighted must modify the subject of the main clause.
Because it is the manager who was delighted, choice C, in which the
company manager appears as the subject, is the best answer.

A, B, D, E - create illogical statements by using it, the decision, the


staff, and a raise,respectively, as the sentence subject.

A , D , E - Use of the passive voice

POSTED BY PRACHI PAREEKH AT 9:34 PM 1 COMMENTS LINKS TO THIS POST

LABELS: MODIFIER ERROR, PASSIVE, PRONOUN ERROR, SUBJECT VERB


AGREEMENT, TENSE

FRIDAY, JUNE 30, 2006

Gmat sentence correction 24 , 25

Gmat Sentence Correction 24 & 25

24). Based on accounts of various ancient writers, scholars have


painted a sketchy picture of the activities of an all-female cult that,
perhaps as early as the sixth century B.C., worshipped a goddess
known in Latin as Bona Dea, "the good goddess."

(A) Based on accounts of various ancient writers

(B) Basing it on various ancient writers' accounts

(C) With accounts of various ancient writers used for a basis

(D) By the accounts of various ancient writers they used

(E) Using accounts of various ancient writers

25). The number of acres destroyed by wildfires, which have become


an ongoing threat due to drought and booming population density,
have increased dramatically over the past several years, prompting
major concern among local politicians.

A). which have become an ongoing threat due to drought and


booming population density, have increased
B). which have become an ongoing threat due to drought and
booming population density, have been increasing

C). which has become an ongoing threat because of drought and


booming population density, has increased

D). which have become an ongoing threat due to drought and


booming population density, has increased

E). which have become an ongoing threat because of drought and


booming population density, has increased

Answers-

24). E is the best choice.

A - Dangling modifier - the introductory clause beginning Based on


modifies scholars, the noun that immediately follows it.
In other words, A says that scholars were based on the accounts of
various ancient writers.

B - is awkward and imprecise - referent of pronoun "it" is not clear.

C, D - are also wordy and awkward.


Further in D - By the accounts... they used is an unidiomatic and
roundabout way of saying that scholars used me accounts.

E - is clear and concise - correctly uses a present participle (or "-ing"


verb) to introduce the modifier describing how the scholars worked.

25). E is the correct choice.

A - The subject of the original sentence is "the number of acres",


which is singular. The main verb, however, is "have increased",
which is plural. We need to find a choice that replaces "have" with
"has". Moreover, the phrase "due to" is incorrect in this context.
"Due to" is a phrase that must describe a noun. "The fire was due to
drought" is correct, but "There was a fire due to drought" is not.
When describing a verb phrase, "because of" is preferable: "There
was a fire because of drought."

B - The plural verb "have been" does not agree with the singular
subject "the number of acres." Additionally, the passive voice "have
been increased" is incorrect. Finally, the phrase "due to drought . . ."
is unidiomatic since "because of" (not "due to") should be used to
modify the verb phrase "have become an ongoing threat."
C - The modifier "which has become an ongoing threat . . ." contains
the singular verb "has" which does not agree with the plural subject
"wildfires."

D - The phrase "due to drought . . ." is unidiomatic since "because of"


(not "due to") should be used to modify the verb phrase "have
become an ongoing threat."

E - Correct - The singular verb "has" agrees with the singular subject
"the number of acres." Additionally, "because of drought . . ." is
properly used to modify the verb phrase "have become an ongoing
threat."
POSTED BY PRACHI PAREEKH AT 12:11 AM 1 COMMENTS LINKS TO THIS
POST
LABELS: IDIOM, MODIFIER ERROR, PRONOUN ERROR, SUBJECT VERB
AGREEMENT, WORDY

WEDNESDAY, JUNE 28, 2006

Gmat sentence correction 22 , 23

22). A recent study has found that within the past few years, many
doctors had elected early retirement rather than face the threats of
lawsuits and the rising costs of malpractice insurance.

(A) had elected early retirement rather than face

(B) had elected early retirement instead of facing

(C) have elected retiring early instead of facing

(D) have elected to retire early rather than facing

(E) have elected to retire early rather than face

23). In metalwork one advantage of adhesive-bonding over spot-


welding is that the contact, and hence the bonding, is effected
continuously over a broad surface instead of a series of regularly
spaced points with no bonding in between.

A) instead of

B) as opposed to
C) in contrast with

D) rather than at

E) as against being at

Answer -

22). Best answer choice is E

A, B - wrong tense 'had elected'


C - here preference is shown so use of 'instead of' is incorrect.
D - violating parallelism - D has parallelism problem. ...elected to
retire....rather than facing....

Hence E is the best choice.

23). D is the best choice, as preference is suggested in the sentence.

A - "instead of " - incorrect - not a substitute


B - "as opposed to" - incorrect - there is no argument
C - "in contrast with" - incorrect
E - "being" - incorrect - as use of 'being'- not entertained in Gmat .
POSTED BY PRACHI PAREEKH AT 11:08 PM 0 COMMENTS LINKS TO THIS
POST
LABELS: PARALLELISM, RATHER THAN VS INSTEAD OF, TENSE

Gmat sentence correction 21

21). Patrice, whom according to the classical musician community is


a virtuoso, plays in a unique style which is all her own but which
also embodies a warmth prevalent during the golden age of violin
playing.

A). whom according to the classical musician community is a


virtuoso, plays in a unique style which is all her own but which also
embodies

B). considered a virtuoso by the community of classical musicians,


plays in a style all her own which at the same time embodies

C). regarded by classical musicians as being a virtuoso, plays in a


unique style all her own yet embodies

D). regarded by classical musicians as a virtuoso, who plays in a


style all her own which at the same time embodies

E). whom the community of classical musicians would consider to be


a virtuoso, plays in a unique style while at the same time embodying

Answer - B is the best choice.Click on the link below to see the


explanation.

Explanation
POSTED BY PRACHI PAREEKH AT 3:56 AM 0 COMMENTS LINKS TO THIS POST

FRIDAY, JUNE 23, 2006

Gmat sentence correction 19 20

19). If anyone at InterCom Financial Advisers would have


anticipated, or even suspected, the impending sale of the Koniko
kelp processing plant, they would have advised owners of Koniko
stock to unload all shares immediately.

(A) If anyone at InterCom Financial Advisers would have


anticipated

(B) Had anyone at InterCom Financial Advisers anticipated

(C) If any people at InterCom Financial Advisers would have


anticipated

(D) If any people at InterCom Financial Advisers had anticipated

(E) If anybody at InterCom Financial Advisers anticipated

20). If the reporter would have known the landlord's side of the
story, she would not have written an article so favorable to the 81-
year-old tenant.

(A) would have known the landlord's side of the story, she would not
have written

(B) would of known the landlord's side of the story, she would not of
written
(C) had known the landlord's side of the story, she would not have
written

(D) had known the landlord's side of the story, she would not have
wrote

(E) knew the landlord's side of the story, she would not have written

Answers -

19). D is the right answer - Correct tense


anyone - singular
any people - plural,
use of " they would .. " in later half of the sentence implies we need
plural .

20). C is the right choice.

Whenever you come across if ... then tense construction in a


sentence,

IF Clause Then Clause

PAST PERFECT ---- WOULD/COULD + HAVE + PARTICIPLE

If Ram had won... he would have given.....

POSTED BY PRACHI PAREEKH AT 3:55 AM 0 COMMENTS LINKS TO THIS POST

LABELS: IF THEN CONSTRUCTION, TENSE

TUESDAY, JUNE 20, 2006

Gmat Sentence Correction 17 , 18

Gmat Sentence Correction 17 & 18

17). Charlotte Perkins Gilman, a late nineteenth-century feminist,


called for urban apartment houses including child-care facilities and
clustered suburban houses including communal eating and social
facilities.

(A) including child-care facilities and clustered suburban houses


including communal eating and social facilities
(B) that included child-care facilities, and for clustered suburban
houses to include communal eating and social facilities

(C) with child-care facilities included and for clustered suburban


houses to include communal eating and social facilities

(D) that included child-care facilities and for clustered suburban


houses with communal eating and social facilities

(E) to include child-care facilities and for clustered suburban houses


with communal eating and social facilities included

18). No one but him could have told them that the king was I.

(A) him could have told them that the king was I.

(B) he could have told them that the king was me.

(C) him could have told them the king was me.

(D) he could have told them that the king was I.

(E) him could had told them that the king was I.

Answer -

17). D is the right choice.

A - AMBIGUITY - including..... - not clear - it is hard to tell whether


Gilman called for urban apartment houses that included child-care
facilities or whether such facilities represent one variety of the
urban apartment houses she wanted to built.

C,E - incorrect - preposition + noun + participle, Gmat almost never


entertains this pattern -- communal eating and social facilities
included.

B - to include - unidiomatic

18). A is the answer.Gmat grammar tip --

"except", "no one but" should be followed by object form of Pronoun.


No one but -- in other words means - except.

Always use subject form of Pronoun after the linking verb.--(Here


linking verb 'was' so we use subject form of pronoun i.e I)

POSTED BY PRACHI PAREEKH AT 10:52 AM 4 COMMENTS LINKS TO THIS


POST
LABELS: PRONOUN ERROR

Gmat Sentence Correction 15 , 16

Gmat Sentence Correction 15 & 16

15). The suspect is facing charges that he illegally published


telephone credit card numbers, intending that they be used by
others to avoid telephone billings.

A. intending that they be used by others to avoid telephone billings.

B. with the intent of others using them to avoid telephone billings.

C. intending that telephone billings could be avoided by others using


them.

D. with the intent of their use by others to avoid telephone billings.

E. with the intent that telephone billings would be avoided by others


through their use.

16). Teratomas are unusual forms of cancer because they are


composed of tissues such as tooth and bone not normally found in
the organ in which the tumor appears.

A. because they are composed of tissues such as tooth and bone

B. because they are composed of tissues like tooth and bone that are

C. because they are composed of tissues, like tooth and bone, tissues

D. in that their composition , tissues such as tooth and bone, is

E. in that they are composed of tissues such as tooth and bone,


tissues
Answer -

15). A is the best choice - The correct idioms are -


1. Intent on verb+ing.
2. intend to verb -- applying 2nd, A is the best choice.

B, D, E -- "numbers" cannot have intent.

C -- use "shall " or "should" instead of "could". Further, use of "them"


is ambiguous as it can either refer to "numbers" or to "billings".

16). E is the best choice.It is a modifier question.(To know about


modifiers click on links below).

A - "not normally found..." modifies "bone"


B - "that.." modifies "bone" again
D - Composition = tissues ? One is singular, the other is plural.

In gmat "like" is used to show similarity whereas "such as" means


example. By this reasoning we straight away eliminate choices B, C.

Further "because" is used to express a simple relationship whereas


"in that" qualifies the previous statement. In this sentence use of
because is wrong, also in Gmat "in that " is preferred over
"because".

http://gmat-grammar.blogspot.com
POSTED BY PRACHI PAREEKH AT 2:43 AM 0 COMMENTS LINKS TO THIS POST

LABELS: IDIOM, LIKE VS SUCH AS, MODIFIER ERROR, PRONOUN ERROR

MONDAY, JUNE 19, 2006

Gmat Sentence Correction 13, 14

SENTENCE CORRECTION 13 & 14

13). In the Renaissance, painters were so impressed with Leonardo


da Vinci that they ignored their own training and designate as a
masterpiece anything he painted.

(A) were so impressed with Leonardo da Vinci that they ignored

(B) were impressed with Leonardo da Vinci to such an extent that


they were to ignore
(C) were so impressed with Leonardo da Vinci as to ignore

(D) were so impressed with Leonardo da Vinci that they had to


ignore

(E) were as impressed with Leonardo da Vinci as to ignore

14). Many of them chiseled from solid rock centuries ago, the
mountainous regions of northern Ethiopia are dotted with hundreds
of monesteries.

(A). Many of them chiseled from solid rock centuries ago, the
mountainous regions of northern Ethiopia are dotted with hundreds
of monesteries.

(B). chiseled from solid rock centuries ago,the mountainous regions


of northern Ethiopia are dotted with hundreds of monesteries.

(C). hundreds of monesteries, many of them chiseled from solid rock


centuries ago, re dotting the mountainous regions of northern
Ethiopia.

(D). The mountainous regions of northern Ethiopia are dotted with


hundreds of monesteries, many of which are chiseled from solid rock
centuries ago.

(E). The mountainous regions of northern Ethiopia are dotted with


hundreds of monesteries,many of them chiseled from solid rock
centuries ago.

Answers :-

13). The correct answer is C.

A - wrong - they ignored..... and designate ... this is incorrect... either


use present tense or past.. not both..

We use 'so as' to mean with the purpose or result - must be followed
by an infinitive. This rule this rule applies in the given sentence.
Painters were impressed... [as a result] they ignored and
designated...

The last part of the sentence uses 'designate'. It's a hint that 'ignore'
should be used in its infinitive form as well (because of the rule and
to preserve parallelism).
Note - GMAT generally prefers "so that" when applicable. However,
because there is an infinitive form "designate" later in the sentence,
you have to use "so as to" to maintain parallelism in verb tense.

14). E is the best choice.

A,B - Wrong because the chiseled ones are monastries and not the
mountain regions.

C - Run-on sentence. It must be many of which.

D - Use of 'are' is incorrect.

E - many of .....ago - absolute phrase - so this is right choice

POSTED BY PRACHI PAREEKH AT 3:00 AM 0 COMMENTS LINKS TO THIS


POST
LABELS: TENSE

TUESDAY, JUNE 13, 2006

Gmat Sentence Correction 11, 12

GMAT SENTENCE CORRECTION -- 11 & 12

11). Three hundred guests is as many as even the most ambitious


host or hostess should invite, and this number of guests is advisable
only when circumstances actually demand it.

A) as many as even

B) so many as even

C) so many that even

D) as many that even

E) even so many that

12). The ideal citeis of Ebenezer Howard, Frank Lloyd Wright, and
Le Corbusier, different from one another as they were, were all
based on a belief that physical order in itself could bring about the
good life.

A). different from one another as they were

B). different one from the other as they were

C). different from each other as every one was

D). differing as each was, one from the other

E). although each differed from

Answer

11). A --- 2 idioms involved here are ---

1- As many as - to the same extent, degree (equal to) -- Ram was as


sad as someone could ever be

2- So... that..- To such an extent/degree that...usually leads to a


conclusion -- Ram was so sad that he decided to leave his job.

In the question :-- Three hundred guests is as many as even the most
ambitious host or hostess should invite.

As can be seen, "even the most ambitious host or hostess should


invite" is not the conclusion of the 300 guests.

Instead,"as many as" will make the proper link between the two
parts => that the 300 guests is equal to what the most ambitious
host should invite.

12). A "as" here is somewhat similar to "even though".


Other catch in this sentence lies in the fact that we often omit the
first "as" when we invert, here it implies -- "as different from one
another as they were" is same as "different from one another as they
were".

I will be posting few sentence correction questions in a day daily,


the explanation and correct answer to each question will be posted
by me at the end of each day, hope this would help all those who are
preparing for Gmat.

1. A “calendar stick” carved centuries ago by the Winnebago tribe


may provide the first evidence that the North American Indians have
developed advanced full-year calendars basing them on systematic
astronomical observation.

(A) that the North American Indians have developed advanced full-
year calendars basing them
(B) of the North American Indians who have developed advanced
full-year calendars and based them
(C) of the development of advanced full-year calendars by North
American Indians, basing them
(D) of the North American Indians and their development of
advanced full-year calendars based
(E) that the North American Indians developed advanced full-year
calendars based

2. A 1972 agreement between Canada and the United States reduced


the amount of phosphates that municipalities had been allowed to
dump into the Great Lakes.

(A) reduced the amount of phosphates that municipalities had been


allowed to dump
(B) reduced the phosphate amount that municipalities had been
dumping
(C) reduces the phosphate amount municipalities have been allowed
to dump
(D) reduced the amount of phosphates that municipalities are
allowed to dump
(E) reduces the amount of phosphates allowed for dumping by
municipalities

3. A collection of 38 poems by Phillis Wheatley, a slave, was


published in the 1770’s, the first book by a Black woman and it was
only the second published by an American woman.

(A) it was only the second published by an American woman


(B) it was only the second that an American woman published
(C) the second one only published by an American woman
(D) the second one only that an American woman published
(E) only the second published by an American woman

4. A common disability in test pilots is hearing impairment, a


consequence of sitting too close to large jet engines for long periods
of time.

(A) a consequence of sitting too close to large jet engines for long
periods of time
(B) a consequence from sitting for long periods of time too near to
large jet engines
(C) a consequence which resulted from sitting too close to large jet
engines for long periods of time
(D) damaged from sitting too near to large jet engines for long
periods of time
(E) damaged because they sat too close to large jet engines for long
periods of time

5. A controversial figure throughout most of his public life, the Black


leader Marcus Garvey advocated that some Blacks return to Africa,
the land that, to him, symbolized the possibility of freedom.

(A) that some Blacks return to Africa, the land that, to him,
symbolized the possibility of freedom
(B) that some Blacks return to the African land symbolizing the
possibility of freedom to him
(C) that some Blacks return to Africa which was the land which
symbolized the possibility of freedom to him
(D) some Black’s returning to Africa which was the land that to him
symbolized the possibility of freedom
(E) some Black’s return to the land symbolizing the possibility of
freedom to him, Africa

6. A fire in an enclosed space burns with the aid of reflected


radiation that preheats the fuel, making ignition much easier and
flames spreading more quickly.

(A) flames spreading


(B) flame spreads
(C) flames are caused to spread
(D) causing flames to spread
(E) causing spreading of the flames

7. A firm that specializes in the analysis of handwriting claims from


a one-page writing sample that it can assess more than three
hundred personality traits, including enthusiasm, imagination, and
ambition.

(A) from a one-page writing sample that it can assess


(B) from a one-page writing sample it has the ability of assessing
(C) the ability, from a one-page writing sample, of assessing
(D) to be able, from a one-page writing sample, to assess
(E) being able to assess, from a one-page writing sample,

8. A huge flying reptile that died out with the dinosaurs some 65
million years ago, the Quetzalcoatlus had a wingspan of 36 feet,
believed to be the largest flying creature the world has ever seen.
(A) believed to be
(B) and that is believed to be
(C) and it is believed to have been
(D) which was, it is believed,
(E) which is believed to be

9. A Labor Department study states that the numbers of women


employed outside the home grew by more than a thirty-five percent
increase in the past decade and accounted for more than sixty-two
percent of the total growth in the civilian work force.

(A) numbers of women employed outside the home grew by more


than a thirty-five percent increase
(B) numbers of women employed outside the home grew more than
thirty-five percent
(C) numbers of women employed outside the home were raised by
more than thirty-five percent
(D) number of women employed outside the home increased by more
than thirty-five percent
(E) number of women employed outside the home was raised by
more than a thirty-five percent increase

10. A large rise in the number of housing starts in the coming year
should boost new construction dollars by several billion dollars,
making the construction industry’s economic health much more
robust than five years ago.

(A) making the construction industry’s economic health much more


robust than five years ago
(B) and make the construction industry’s economic health much
more robust than five years ago
(C) making the construction industry’s economic health much more
robust than it was five years ago
(D) to make the construction industry’s economic health much more
robust than five years ago
(E) in making the construction industry’s economic health much
more robust than it as five years ago

Answers

1.) E is the best choice as it only maintains parallelism -- carved...


developed....based

2.) OA - D.
Since the dumping continued after the date of the agreement, the
past perfect verb had been allowed in choice A is incorrect. The verb
should instead be the present are allowed(if the agreement remained
in effect when the sentence was written) or the past were allowed(if
the agreement was no longer in effect when the sentence was
written). B changes the meaning of the sentence by stating that – the
amount of phosphate that municipalities were dumping whereas
stem only talks about the limit and not whether they were up to the
limit.C, E use wrong tense reduces, should be the past tense reduced.
Further allowed for dumping in E is unidiomatic. Also the phrase
amount of phosphate is clear and idiomatically correct, whereas
phosphate amount is not idiomatic

3). The best choice is E, Here parallelism has to be maintained


between two nouns and not two sentences, thus options A and B are
ruled out.Also in choices A and B use of it denotes redundancy. This
leaves us with C, D, and E. Choices C and D are wordy, also
according to the grammar rule one should put only as close as
possible to the word it is suppose to modify.Further choice D is
depicting incorrect placing of active and passive voice constructions
in the sentence.

4). A is the best choice.

5). A is the best answer. D, E are ruled out because of use of


apostrophe which is changing the expression to noun form. Option B
is unclear and suffers from ambiguity and in option C the referent of
which is not clear.

6). D is the best choice.

7). D is the best answer. It correctly uses an infinitive to connect the


verb "claims" with the firm's assertion : claims to be able .....to
assess....All of the other choices use ungrammatical or unclear
constructions after claims. Choices A and B present clauses that
should be introduced by "claims that". In A, placing that after
sample rather than after claims produces the unintended statement
that the claim itself is made on the basis of a single one - page
writing sample.Also in B, the ability of assessing is
unidiomatic.Choice C repeats this second fault and uses the
unidiomatic claims the ability.Choice E uses the ungrammatical
claims being able to assess.

8). C is the best choice.


A - incorrect - participial phrase is incorrectly referring to 36 feet.
Further wrongly suggests that Quetzalcoatlus still exists though the
real meaning conveyed here is that it is still considered the largest
flying creature (present perfect)
B - incorrect - "that" is modifying "a wingspan", instead of
modifying "Quetzalcoatlus".
D, E - incorrect - "which" modifying "a wingspan", instead of
modifying "Quetzalcoatlus"

9). D is the best choice.Because a count of women employed outside


the home at any given time will be expressed by a single number, the
use of the plural noun numbers in choices A, B, and C is illogical. In
A, the phrase grew by more than a thirty-five percent increase is
redundant and wordy, since the sense of increase is implicit in the
verb grew. In C and E, the passive verb forms were raised and was
raised are inappropriate because there is no identifiable agent
responsible for the raising of the number of women employed. In
choice E, was raised by ... increase is redundant. Choice D, which
presents the comparison logically and idiomatically, is the best
answer.

10). C is the best answer.Wrong comparison - health to five years


ago eliminate option A, B, and D .In option E in making is not
necessary, thus making it unnecessarily wordy.

You might also like